Đến nội dung

shinichigl nội dung

Có 120 mục bởi shinichigl (Tìm giới hạn từ 29-04-2020)



Sắp theo                Sắp xếp  

#688685 $x^{y}=(y+1)^{x}$

Đã gửi bởi shinichigl on 26-07-2017 - 10:58 trong Phương trình - Hệ phương trình - Bất phương trình

Đặt $\sqrt{y+1}=b$; $\sqrt{-4x^{2}+18x-20}=a$

Từ đó ta có: $b> 0$; $a\geq 0$; $2\leq x\leq 2.5$

Từ phương trình đầu tiên ta có:

$1+a=b-\frac{4}{a^{2}+4}$

$\Leftrightarrow b=\frac{a^{3}+a^{2}+4a+8}{a^{2}+4}$

$\Rightarrow b\geq \frac{8}{4}=2$ hay $b^{2}\geq 4$

(Do hàm số $f(a)=\frac{a^{3}+a^{2}+4a+8}{a^{2}+4}$ là hàm đồng biến trên tập xác định (tính đạo hàm ra sẽ thấy))

Từ phương trình thứ hai ta có:

$x^{b^{2}}=\left ( b^{2} \right )^{x}$

$\Leftrightarrow ln\left ( x^{b^{2}} \right )=ln\left ( \left ( b^{2} \right )^{x} \right )$

$\Leftrightarrow b^{2}ln(x)=xln(b^{2})$

$\Leftrightarrow \frac{ln(x)}{x}=\frac{ln\left ( b^{2} \right )}{b^{2}}$

Xét $f(t)=\frac{ln(t)}{t}$ $(t> 0)$

$f'(t)=\frac{1-ln(t)}{t^{2}}$

Từ đó ta có:

Với $0<t<e$ thì $f(t)$ đồng biến

Với $t>e$ thì $f(t)$ nghịch biến

Suy ra: 

$\frac{ln(x)}{x}\geq \frac{ln(2)}{2}$ và $\frac{ln(b^{2})}{b^{2}}\leq  \frac{ln(4)}{4}=\frac{ln(2)}{2}$

Đẳng thức xảy ra khi: $b=2$; $x=2$ hay $y=3$; $x=2$ (*)

Thay (*) vào phương trình thứ nhất ta thấy thõa mãn

Vậy hệ phương trình có 1 nghiệm là $\left (x;y \right )=\left ( 2;3 \right )$




#688617 $2(\sqrt{1-5x}-\sqrt{x}-\sqrt{x-...

Đã gửi bởi shinichigl on 25-07-2017 - 13:09 trong Phương trình - hệ phương trình - bất phương trình

nham nghiem kieu gi vay ban

Mình dùng máy CASIO để solve nghiệm của nó. Lúc mà thay nghiệm vào thì thấy $\sqrt{1-5x}=\sqrt{x-x^{2}}$. Từ đó tìm ra




#688575 $2(\sqrt{1-5x}-\sqrt{x}-\sqrt{x-...

Đã gửi bởi shinichigl on 24-07-2017 - 22:28 trong Phương trình - hệ phương trình - bất phương trình

Điều kiện: $0\leq 0\leq \frac{1}{5}$ (1)

$2\left ( \sqrt{1-5x}-\sqrt{x} -\sqrt{x-x^{2}}\right )=x-1$

$\Leftrightarrow 2\left ( \sqrt{1-5x} -\sqrt{x-x^{2}}\right )=2\sqrt{x}-\left ( 1-x \right )$

$\Leftrightarrow 2\frac{x^{2}-6x+1}{\sqrt{1-5x}+\sqrt{x-x^{2}}}=(-1)\frac{x^{2}-6x+1}{2\sqrt{x}+\left ( 1-x \right )}$

$\Leftrightarrow x^{2}-6x+1=0$

(Vì phương trình $\frac{2}{\sqrt{1-5x}+\sqrt{x-x^{2}}}=\frac{-1}{2\sqrt{x}+\left ( 1-x \right )}$ vô nghiệm (VT>0 và VP<0))

$\Leftrightarrow x=3-2\sqrt{2}$ (do (1))




#688464 GPT: $\sqrt[3]{2x^3+6}=x+\sqrt{x^2-3x+3}...

Đã gửi bởi shinichigl on 23-07-2017 - 21:50 trong Phương trình - Hệ phương trình - Bất phương trình

Đặt $\sqrt{x^{2}-3x+3}=b$ (1) ($b\geq 0$)

Ta có hệ phương trình

$\left\{\begin{matrix} 2x^{3}+6=\left ( x+b \right )^{3}\\ x^{2}-3x+3=b^{2} \end{matrix}\right.$

$\Leftrightarrow \left\{\begin{matrix} \left ( x-b \right )^{3}+6=6xb^{2}\\ x^{2}-3x+3=b^{2} \end{matrix}\right.$
$\Leftrightarrow \left\{\begin{matrix} \left ( x-b \right )^{3}+6=6x\left ( x^{2}-3x+3 \right )\\ x^{2}-3x+3=b^{2} \end{matrix}\right.$
$\Leftrightarrow \left\{\begin{matrix} \left ( x-b \right )^{3}=6\left ( x-1 \right )^{3}\\ x^{2}-3x+3=b^{2} \end{matrix}\right.$
$\Rightarrow b=\left ( 1-\sqrt[3]{6} \right )x+\sqrt[3]{6}$ (2)
Thay (2) vào (1) ta tìm được 2 nghiệm là
$x_{1}=1;x_{2}=\frac{\sqrt[3]{36}-3}{\sqrt[3]{36}-2\sqrt[3]{6}}$



#687653 $\sum \frac{a}{b}\geq \frac{3}{2}\left ( a+b+c-...

Đã gửi bởi shinichigl on 15-07-2017 - 22:17 trong Bất đẳng thức và cực trị

Trong 3 số (a-1), (b-1), (c-1) luôn luôn có 2 số cùng không âm hoặc không dương. Giả sử (a-1) và (b-1) cùng không âm hoặc cùng không dương
Khi đó ta có: (a-1)(b-1)$\geq 0$ (*)

Từ đề bài: $a=\frac{1}{bc}$. Ta cần chứng minh $\frac{1}{b^{2}c}+\frac{b}{c}+bc^{2}\geq \frac{3}{2}\left ( \frac{1}{bc}+b+c-1 \right )$

Ta lại có:

$\frac{1}{b^{2}c}+\frac{b}{c}+bc^{2}=\left ( \frac{1}{2b^{2}c}+\frac{b}{c} \right )+\left ( \frac{1}{2b^{2}c}+bc^{2} \right )$

$\geq\frac{3}{2} \left ( \sqrt[3]{\frac{1}{b^{2}c}.\left ( \frac{b}{c} \right )^{2}}+\sqrt[3]{\frac{1}{b^{2}c}.\left ( bc^{2} \right )^{2}} \right )$

$=\frac{3}{2}\left ( \frac{1}{c}+c \right )$

Mặt khác: $\frac{1}{c}+c\geq \frac{1}{bc}+b+c-1\Leftrightarrow \frac{1}{c}+1-\frac{1}{bc}-b\geq 0\Leftrightarrow \left ( b-1 \right )\left ( \frac{1}{bc}-1 \right )\geq 0\Leftrightarrow \left ( b-1 \right )\left ( a-1 \right )\geq 0$ Đúng theo (*)
Vậy bđt được chứng minh



#551488 Đề thi Olympic 30/4 chính thức lần thứ XXI năm 2015 (lớp 11)

Đã gửi bởi shinichigl on 05-04-2015 - 00:45 trong Thi HSG cấp Tỉnh, Thành phố. Olympic 30-4. Đề thi và kiểm tra đội tuyển các cấp.

Bài 6 (3 điểm)

Tại ba đỉnh $A,B,C$ của một tam giác $ABC$, người ta viết các số $a,b,c$. Người ta thực hiện phép biến đổi sau : Nếu mỗi bộ trước là $(x,y,z)$ thì sau đó ta thay bởi bộ $(x+y-2z,y+z-2x,z+x-2y)$. Chứng minh rằng sau một số lần biến đổi sẽ tồn tại một bộ ba số mà ít nhất một trong ba số của nó không nhỏ hơn $2015$.

 

Bài 6 (Mình nghĩ cần phải thêm điều kiện ít nhất trong 3 số $a$, $b$, $c$ phải có một số khác $0$)

Sau lần biến đổi thứ nhất thì tổng của 3 số $a$, $b$, $c$ bằng $0$ nên ta có $a_{n}+b_{n}+c_{n}=0,n\geq 1$

($n$ là số lần biến đổi của bộ $\left ( a,b,c \right )$)

Ta xét tổng $S_{n}=a_{n}^{2}+b_{n}^{2}+c_{n}^{2}$

Khi đó $S_{n+1}=\left ( a_{n}+b_{n}-2c_{n} \right )_{n}^{2}+\left ( b_{n}+c_{n}-2a_{n} \right )^{2}+\left ( c_{n}+a_{n}-2b_{n} \right )^{2}=6S_{n}-6\left ( a_{n}b_{n}+b_{n}c_{n}+c_{n}a_{n} \right )$

Mặt khác $a_{n}+b_{n}+c_{n}=0$ nên $a_{n}^{2}+b_{n}^{2}+c_{n}^{2}=-2\left ( a_{n}b_{n}+b_{n}c_{n}+c_{n}a_{n} \right )$

Suy ra $S_{n+1}=3S_{n}=3^{n}S_{1}$

Ta có $\left ( \left | a_{n} \right |+\left | b_{n} \right |+\left | c_{n} \right | \right )^{2}> a_{n}^{2}+b_{n}^{2}+c_{n}^{2}$

Suy ra $\left | a_{n} \right |+\left | b_{n} \right |+\left | c_{n} \right |> 3^{\frac{n-1}{2}}\sqrt{S_{1}}$

(do trong 3 số $a$, $b$, $c$ phải có ít nhất một số khác $0$ nên $S_{1}>0$)

Giả sử $\left | a_{n} \right |$ là số lớn nhất trong 3 số $\left | a_{n} \right |$, $\left | b_{n} \right |$, $\left | c_{n} \right |$

Khi đó $\left | a_{n} \right |> 3^{\frac{n-3}{2}}\sqrt{S_{1}}$. Nếu $a_{n}>0$ thì $a_{n}> 3^{\frac{n-3}{2}}\sqrt{S_{1}}>\frac{3^{\frac{n-3}{2}}\sqrt{S_{1}}}{2}$

Nếu $a_{n}<0$ thì $a_{n}<- 3^{\frac{n-3}{2}}\sqrt{S_{1}}$ hay $b_{n}+c_{n}> 3^{\frac{n-3}{2}}\sqrt{S_{1}}$

Suy ra một trong 2 số $b_{n}$, $c_{n}$ phải lớn hơn $\frac{3^{\frac{n-3}{2}}\sqrt{S_{1}}}{2}$

Vậy một trong 3 số $a_{n}$, $b_{n}$, $c_{n}$ phải lớn hơn $\frac{3^{\frac{n-3}{2}}\sqrt{S_{1}}}{2}$.

Từ đó với $n$ đủ lớn thì $\frac{3^{\frac{n-3}{2}}\sqrt{S_{1}}}{2}\geq 2015$

Vậy ta có điều phải chứng minh




#549749 $\sum\frac{x}{x^2+1}\leq \frac...

Đã gửi bởi shinichigl on 27-03-2015 - 16:11 trong Bất đẳng thức - Cực trị

Cách khác:

Áp dụng $AM-GM$, ta có:

$x^2+\frac{1}{9}\geq 2\sqrt{x^2.\frac{1}{9}}=\frac{2}{3}x$

Do đó, $\sum\frac{x}{x^2+1}\leq\sum\frac{x}{\frac{2}{3}x+\frac{8}{9}}$

$\Leftrightarrow\sum\frac{x}{x^2+1}\leq\sum\frac{9x}{6x+8}$

$\Leftrightarrow\frac{2}{3}\sum\frac{x}{x^2+1}\leq\sum\frac{6x}{6x+8}=\sum 1-\frac{8}{6x+8}\leq 3-8\frac{9}{\sum 6x+8}=3-\frac{72}{6(x+y+z)+24}$

$\Leftrightarrow\sum\frac{x}{x^2+1}\leq\frac{3}{2}(3-\frac{72}{30})=\frac{9}{10}$

x,y,z là số thực nên đâu có chỗ này đâu bạn




#546340 $ n\mid k^{a}-k^{b} $

Đã gửi bởi shinichigl on 26-02-2015 - 17:42 trong Số học

Ta có $k^{a}-k^{b}\vdots n,\forall k$ nên $k^{\left | a-b \right |}-1\vdots n,\forall k$.Từ đó ta cũng có $\left (2k  \right )^{\left | a-b \right |}-1\vdots n,\forall k$.

Suy ra $2^{\left | a-b \right |}-1\vdots n$. Do $0\leq a,b\leq 9$ nên $0\leq \left | a-b \right |\leq 9$.

Từ đó lần lượt thay các giá trị của $\left | a-b \right |$ ta sẽ tìm được nghiệm.

Vậy các nghiệm thõa mãn đề bài là $15$ và $\overline{aa}$.




#545877 Chứng minh $(a^2+b^2, c^2+d^2)>1$

Đã gửi bởi shinichigl on 24-02-2015 - 16:31 trong Số học

Cho các số nguyên dương a, b, c, d sao cho $(ac+bd) \vdots (a^2+b^2)$.  Chứng minh $(a^2+b^2, c^2+d^2)>1$ 

Từ giả thiết ta có $a\left ( ac+bd \right )\vdots \left ( a^{2}+b^{2} \right )$. Suy ra $c\left ( a^{2}+b^{2} \right )+b\left ( ad-bc \right )\vdots \left ( a^{2}+b^{2} \right )$.

Như vậy $b\left ( ad-bc \right )\vdots \left ( a^{2}+b^{2} \right )$. Suy ra $b\left ( adc-bc^{2} \right )\vdots \left ( a^{2}+b^{2} \right )$ (1).

Cũng từ giả thiết đề bài ta có $b\left ( adc+bd^{2} \right )\vdots \left ( a^{2}+b^{2} \right )$ (2).

Từ (1) và (2) ta có $b^{2}\left ( c^{2}+d^{2} \right )\vdots \left ( a^{2}+b^{2} \right )$.

Nếu $\left ( a^{2}+b^{2};c^{2}+d^{2} \right )=1$ thì $b^{2}\vdots \left ( a^{2}+b^{2} \right )$ (vô lý).

Vậy $\left ( a^{2}+b^{2};c^{2}+d^{2} \right )>1$.

Nhận xét: Nếu $\left ( a;b \right )=1$ thì $\left ( c^{2}+d^{2} \right )\vdots \left ( a^{2}+b^{2} \right )$.




#545860 Chứng minh hệ phương trình đồng dư có nghiệm

Đã gửi bởi shinichigl on 24-02-2015 - 15:14 trong Số học

Chứng minh rằng hệ các đồng dư

$\begin{cases} x\equiv a_1 (\textit{mod }m_1) \\ x\equiv a_2 (\textit{mod }m_2)\\... \\x\equiv a_r (\textit{mod }m_r)\end{cases}$

có nghiệm nếu và chỉ nếu $(m_i,m_j)\mid (a_i-a_j)$ với mọi cặp số nguyên $(i,j)$, $1\le i\le j \le r$. Chỉ ra rằng nếu nghiệm tồn tại thì là nghiệm duy nhất môđulô $[m_1,m_2,...,m_r]$.

Đây là mở rộng của Định lý phần dư Trung Hoa, bạn có thể tìm cách chứng minh trong cuốn SỐ HỌC của VMF




#545857 $a^n-1$ không chia hết cho $n$

Đã gửi bởi shinichigl on 24-02-2015 - 14:40 trong Số học

Cho $n$ là số nguyên dương lẻ và lớn hơn $1$. Chứng minh rằng với mọi số $a=2^k+1$ với $k$ là số nguyên dương thì 

$a^n-1$ không chia hết cho $n$

ch: chia hết cho; kch: không chia hết cho.

Giả sử $n$ là số nguyên dương lẻ nhỏ nhất thõa mãn $a^{n}-1$ ch $n$.

Nếu $\left ( a;n \right )=f>1$ thì $a^{n}-1$ kch $f$. Suy ra $a^{n}-1$ kch $n$.

Vậy $\left ( a;n \right )=1$. Theo tiêu chuẩn Euler ta có $a^{\varphi \left ( n \right )}-1$ ch $n$.

Mặt khác $\left (a^{n}-1;a^{\varphi \left ( n \right )}-1  \right )=a^{d}-1$, với $\left ( n;\varphi \left ( n \right ) \right )=d$.

Từ đó $a^{d}-1$ ch $n$. Nếu $d=1$ thì $a-1=2^{k}$ kch $n$ (do $n$ là số lẻ).

Vậy $d>1$. Suy ra $a^{d}-1$ ch $d$ (do $\left ( n;\varphi \left ( n \right ) \right )=d$).

Do $\left ( n;\varphi \left ( n \right ) \right )=d$ nên $1<d\leq \varphi \left ( n \right )<n$

Điều này vô lý với định nghĩa $n$ ban đầu.

Vậy $a^{n}-1$ kch $n$

Chú ý $\left ( a^{m}-1;a^{n}-1 \right )=a^{d}-1$, với $d=\left ( m;n \right )$

Thật vậy,

Giả sử $m>n$ và $m=qn+r$.

Ta có $\left ( a^{m}-1;a^{n}-1 \right )=\left ( a^{qn}-1+a^{qn}\left ( a^{r}-1 \right );a^{n}-1 \right )=\left ( a^{r}-1;a^{n}-1 \right )$

(do $a^{m}-1$ và $a^{n}-1$ kch $a$).

Từ đó ta có $\left ( m;n \right )=\left ( n;r \right )\Leftrightarrow \left ( a^{m}-1;a^{n}-1 \right )=\left ( a^{n}-1;a^{r}-1 \right )$.

Vậy theo thuật chia Euclide thì ta có điều phải chứng minh.




#545853 Đề thi chọn đội tuyển HSG QG Hà Nội năm học 2014-2015

Đã gửi bởi shinichigl on 24-02-2015 - 14:10 trong Thi HSG cấp Tỉnh, Thành phố. Olympic 30-4. Đề thi và kiểm tra đội tuyển các cấp.

Đề thi chọn đột tuyển Hà Nội vòng 2 năm học 2014-2015

Bài 2

Nếu tồn tại $y_{0}$ sao cho $f\left (y_{0}  \right )>1$ thì thay $x$ bởi $\frac{y_{0}}{f\left ( y_{0} \right )-1}$ ta có

$f\left ( \frac{y_{0}f\left ( y_{0} \right )}{f\left ( y_{0} \right )-1} \right )f\left ( y_{0} \right )=f\left ( \frac{y_{0}f\left ( y_{0} \right )}{f\left ( y_{0} \right )-1} \right )$. Suy ra $f\left ( y_{0} \right )=1$ (vô lý). Vậy $f\left ( y \right )\leq 1,\forall y>0$

Từ đó ta có $f\left ( y \right )\geq f\left ( x+y \right ),\forall x>0,y>0$. Suy ra $f$ là hàm không tăng

Nếu tồn tại $y_{1}$ sao cho $f\left (y_{1}  \right )=1$ thì thay $y$ bởi $y_{1}$ ta có $f\left ( x \right )=f\left ( x+y_{1} \right ),\forall x>0$

Do $f$ là hàm không tăng và tuần hoàn nên $f$ là hàm hằng. Vậy $f\left ( y \right )=1,\forall y>0$

Ta xét $f\left ( y \right )< 1,\forall y>0$. Suy ra $f$ nghịch biến hay $f$ là hàm đơn ánh.

Thay $x$ bởi $\frac{x}{f\left ( y \right )}$ ta có $f\left ( x \right )f\left ( y \right )=f\left ( \frac{x}{f\left ( y \right )}+y \right ),\forall x>0,y>0$

Hoán đổi vị trí $x$ và $y$ ta được $f\left (\frac{x}{f\left ( y \right )}+y  \right )=f\left ( \frac{y}{f\left ( x \right )}+x \right ),\forall x>0,y>0$

Suy ra $\frac{x}{f\left ( y \right )}+y=\frac{y}{f\left ( x \right )}+x,\forall x>0,y>0$

Như vậy $\frac{1-f\left ( y \right )}{yf\left ( y \right )}=k,\forall y>0$ ($k$ là hằng số, $k\neq 0$)

Suy ra $f\left ( y \right )=\frac{1}{1+ky},\forall y>0$

Thử lại 2 hàm trên, ta thấy đều thõa mãn đề bài.

Vậy $f\left ( y \right )\equiv 1$, $f\left ( y \right )\equiv \frac{1}{1+ky}$




#541175 VMO 2015

Đã gửi bởi shinichigl on 18-01-2015 - 13:15 trong Thi HSG Quốc gia và Quốc tế

 

NGÀY 2
Bài 5: (7,0 điểm) Cho $(f_n(x))$ là dãy đa thức xác định bởi:
$f_0(x)=2,f_1(x)=3x,f_n(x)=3xf_{n-1}(x)+(1-x-2x^2)f_{n-2}(x)$ với mọi $n\ge 2$. 
Tìm tất cả các số nguyên dương $n$ để $f_n(x)$ chia hết cho đa thức $x^3-x^2+x$.

Áp dụng phương trình sai phân bậc hai, ta tìm được công thức tổng quát: $f_{n}(x)=(2x-1)^{n}+(x+1)^{n}$ (*)

Thay $x=5$ vào (*) ta được $f_{n}(5)=9^{n}+6^{n}$

Ta sẽ tìm $n$ để $f_{n}(5)=9^{n}+6^{n}$ chia hết cho $5^{3}-5^{2}+5=105$

Ta có $9^{n}+6^{n}=3^{n}\left ( 3^{n}+2^{n} \right )$ và $105=3.5.7$

Suy ra ta chỉ cần tìm $n$ sao cho $3^{n}+2^{n}$ chia hết cho $7$

Đặt $n=6k+r$ ($r\in \left \{ 0,1,2,3,4,5 \right \}$)

Từ đó ta có $3^{n}+2^{n}=3^{6k+r}+2^{6k+r}=729^{k}.3^{r}+64^{k}.3^{r}$

Suy ra $3^{n}+2^{n}\equiv 3^{r}+2^{r}(mod7)$

Lần lượt thay $r= 0,1,2,3,4,5$ ta thấy chỉ có $r=3$ thõa mãn $3^{n}+2^{n}\equiv 0(mod7)$

Tiếp theo ta sẽ chứng minh $n=6k+3$ thõa mãn (bằng quy nạp theo $k$)

Thật vậy,

Với $k=0$ ($n=3$) ta có $f_{n}=f_{3}=9x^{3}-9x^{2}+9x$ chia hết cho $x^{3}-x^{2}+x$

Giả sử $k=a$ thõa mãn, ta sẽ chứng minh $k=a+1$ cũng thõa mãn

Ta có 

$(2x-1)^{6a+9}+(x+1)^{6a+9}=\left [ (2x-1)^{6a+3}+(x+1)^{6a+3} \right ]\left [ (2x-1)^{6}+(x+1)^{6} \right ]-(2x-1)^{6}(x+1)^{6}\left [ (2x-1)^{6a-3}+(x+1)^{6a-3} \right ]$

chia hết cho $x^{3}-x^{2}+x$

Vậy theo nguyên lý quy nạp ta có $n=6k+3$, với mọi $k$ nguyên không âm, thõa mãn đề bài




#541172 $y^3=3x^2+3x+7$

Đã gửi bởi shinichigl on 18-01-2015 - 12:04 trong Các bài toán Đại số khác

Cho $x,y \in \mathbb{Z}$.tìm $x,y$ tm:

$y^3=3x^2+3x+7$

Từ giả thiết ta suy ra $y\equiv 1(mod3)$. Đặt $y=3k+1$

Từ đó ta có $(3k+1)^{3}=3x^{2}+3x+7\Leftrightarrow 9k^{3}+9k^{2}+3k=x^{2}+x+2$

Suy ra $x^{2}+x+2\equiv 0(mod3)$

Mặt khác $x^{2}+x+2\equiv 0(mod3)$

          $\Leftrightarrow 4x^{2}+4x+8\equiv 0(mod3)$

          $\Leftrightarrow (2x+1)^{2}+7\equiv 0(mod3)$

          $\Leftrightarrow (2x+1)^{2}+1\equiv 0(mod3)$ (*)

Ta có $(2x+1)^{2}\equiv 0;1(mod3)$ (do một số chính phương thì chia 3 dư 0 hoặc 1)

      $\Leftrightarrow (2x+1)^{2}+1\equiv 1;2(mod3)$. Mâu thuẫn với (*)

Vậy phương trình trên không có nghiệm nguyên trên $\mathbb{Z}$




#535485 $f(x+f(x)+2y)=2x+2f(f(y))$

Đã gửi bởi shinichigl on 30-11-2014 - 10:29 trong Phương trình hàm

Tìm hàm $f:\mathbb{Q}\rightarrow \mathbb{Q}$ thỏa mãn: $f(x+f(x)+2y)=2x+2f(f(y))$ (*)

Trong (*) thay $x=y$ ta được: $f(f(0))=2f((0)) \Rightarrow f(f(0))=0$

Trong (*) thay $y=0$ ta được: $f(x+f(x))=2x,\forall x\in \mathbb{Q}$ (1) $\Rightarrow$ $f$ toàn ánh

Do $f$ toàn ánh nên tồn tại $a$ sao cho $f(a)=0$

Thay $x=a$ vào (1) ta được: $f(a)=2a \Rightarrow f(0)=0$

Trong (*) thay $x=0$ ta được: $f(2y)=2f(f(y)), \forall y \in \mathbb{Q}$ (2)

Trong (1) cho $x=1$ ta được $f(1+f(1))=2$

Do f toàn ánh nên tồn tại $c$ sao cho $f\left ( c \right )=1+f(1)$

Từ (*) và (2) ta có: $f(x+f(x)+2y)=2x+f(2y), \forall x,y \in \mathbb{Q}$

$\Rightarrow f(x+f(x)+y)=2x+f(y), \forall x,y \in \mathbb{Q}$ (3)

Trong (3) thay $x=1$ và $y=-1-f(1)$ ta được: $f(-1-f(1))=-2$

Trong (3) thay $x=c$ và $y=-1-f(1)$ ta được: $f\left ( c \right )=2c-2 \Rightarrow f(1)+3=2c$

Trong (2) thay $y=c$ ta được: $f(2c)=4$

Trong (*) thay $x=y=1$ ta được: $f(3+f(1))=2+2f(f(1)) \Rightarrow f(f(1))=1$

Trong (1) thay $x=f(1)$ ta được: $f(f(1)+1)=2f(1) \Rightarrow f(1)=1$

Trong (3) thay $x=1$ ta được: $f(2+y)=2+f(y),\forall y\in \mathbb{Q}$ (4)

Từ (4) bằng quy nạp ta chứng minh được: $f(x)=x,\forall x\in \mathbb{Z}$

Từ (3) bằng quy nạp ta chứng minh được: $f\left ( n\left ( x+f(x) \right ) \right )=nf\left ( x+f(x) \right )=2nx,\forall n\in \mathbb{Z},x\in \mathbb{Q}$ (5)

Trong (5) thay $x=\frac{1}{2n}$ ta được: $f\left ( n\left ( \frac{1}{2n}+f\left ( \frac{1}{2n} \right ) \right ) \right )=1$

Đặt $n\left ( \frac{1}{2n}+f\left ( \frac{1}{2n} \right ) \right )=b$

Trong (3) thay $x=b$ và $y=-f(b)$ ta được: $f\left ( b+f(b)-f(b) \right )=2b+f(-f(b))\Rightarrow b=1$

$\Rightarrow f\left ( \frac{1}{2n} \right )=\frac{1}{2n},\forall n\in \mathbb{Z}^{*}$

Trong (5) thay $x$ bởi $\frac{1}{2m}$ ta được: $f\left ( n\left ( \frac{1}{2m}+f\left ( \frac{1}{2m} \right ) \right ) \right )=\frac{n}{m},\forall n\in \mathbb{Z},m\in \mathbb{Z}^{*}$

$\Rightarrow f\left ( \frac{n}{m} \right )=\frac{n}{m},\forall n\in \mathbb{Z},m\in \mathbb{Z}^{*}$

$\Rightarrow f(x)=x,\forall x\in \mathbb{Q}$




#533852 $f(yf(x+y)+f(x))= 4x+2yf(x+y)$

Đã gửi bởi shinichigl on 20-11-2014 - 00:48 trong Phương trình hàm

$f:R\rightarrow R$ thoả mãn $f(yf(x+y)+f(x))= 4x+2yf(x+y) (1)$ 

Trong (1) cho $y=0$ ta được $f(f(x))=4x,\forall x\in \mathbb{R}$ (2), suy ra $f$ song ánh

Từ đó tồn tại duy nhất $c$ sao cho $f\left ( c \right )=2$

Trong (1) thay $y$ bởi $c-x$ ta có

$f((c-x)f\left ( c \right )+f(x))=4x+2(c-x)f\left ( c \right ),\forall x\in \mathbb{R}
\Rightarrow f(2c-2x+f(x))=4c,\forall x\in \mathbb{R}$

Trong (2) cho $x=c$ ta được $f(f\left ( c \right ))=4c=f(2)$

suy ra $2c-2x+f(x)=2,\forall x\in \mathbb{R}
\Rightarrow f(x)=2x+2-2c,\forall x\in \mathbb{R}$

Từ đó ta tìm được $f(x)=2x,\forall x\in \mathbb{R}
$




#529423 Chọn đội tuyển QG tỉnh Gia Lai 2014-2015

Đã gửi bởi shinichigl on 18-10-2014 - 20:47 trong Thi HSG cấp Tỉnh, Thành phố. Olympic 30-4. Đề thi và kiểm tra đội tuyển các cấp.

Câu 5:

Ta sẽ chứng minh $\mathbb{N}^{*}\subset A$

Điều này tương đương với việc chứng minh $\forall x\in \mathbb{N}^{*}$ thì $x\in A$

Theo giả thiết $\left \{ 1;2;3 \right \}\in A$

Do $2;3\in A$ nên $1+2.3=7\in A$

Do $2;7\in A$ nên $1+2.7=15\in A$ $\Rightarrow 5\in A$

Do $5;3\in A$ nên $1+5.3=16\in A$ $\Rightarrow 4\in A$

Do $5;7\in A$ nên $1+5.7=36\in A$ $\Rightarrow 6\in A$

Từ đó ta có $\left \{ 1;2;3;4;5;6;7 \right \}\in A$

Giả sử $k\in A$

Trường hợp 1: $k$ chẵn

Khi đó ta có $\frac{k}{2}\in \mathbb{N}^{*}$

Mà $\frac{k}{2}<k$ nên $\frac{k}{2}\in A$

Suy ra $1+2.\frac{k}{2}=k+1\in A$

Trường hợp 2: $k$ lẻ

Khi đó ta có $\frac{k+1}{2}\in \mathbb{N}^{*}$

Mà $\frac{k+1}{2}<k$ nên $\frac{k+1}{2}\in A$

Suy ra $1+2.\frac{k+1}{2}=k+2\in A$

Suy ra $1+\left ( k+2 \right )k=\left ( k+1 \right )^{2}\in A$ $\Rightarrow k+1\in A$

Từ hai trường hợp trên ta có $k+1\in A$

Vậy theo nguyên lí quy nạp ta chứng minh được $\forall x\in \mathbb{N}^{*}$ thì $x\in A$

Từ đó ta có $\mathbb{N}^{*}\subset A$

Mà theo giả thiết thì ta có $A\subset \mathbb{N}^{*}$

Suy ra $A=\mathbb{N}^{*}$

Vậy $2013^{2014}\in A$

 

@supermember: giỏi quá (clap)




#529417 Chọn đội tuyển QG tỉnh Gia Lai 2014-2015

Đã gửi bởi shinichigl on 18-10-2014 - 20:13 trong Thi HSG cấp Tỉnh, Thành phố. Olympic 30-4. Đề thi và kiểm tra đội tuyển các cấp.

Câu 4:

Rõ ràng $n>3$

Ta xét tập $A=\left \{ b_{1},b_{2},...,b_{n} \right \}$

 với $b_{i}=\frac{x_{i}}{2^{k}},\forall i\in \left [ 1,n \right ]$

 $k$ là số thõa mãn $x_{i}\vdots 2^{k},\forall i\in \left [ 1,n \right ]$ và $\exists j\in \left [ 1,n \right ]$ sao cho $x_{j}$ không chia hết $2^{k+1}$

Từ đó, ta có tập $A$ cũng thõa mãn tính chất của tập $X$

Theo giả thiết, khi ta bỏ phần tử $b_{n}$ ra khỏi tập $A$ thì tập $A^{'}=\left \{ b_{1},b_{2},...,b_{n-1} \right \}$ có thể phân hoạch thành hai tập con khác rỗng sao cho tổng tất cả các phần tử trong mỗi tập con đó bằng nhau.

Suy ra tổng $b_{1}+b_{2}+...+b_{n-1}$ là một số chẵn

Tương tự ta có tổng $b_{1}+b_{2}+...+b_{n-2}+b_{n};$

                                                 ......

                                 $b_{2}+b_{3}+...+b_{n}$

là một số chẵn

Suy ra $b_{1},b_{2},...,b_{n}$ cùng chẵn hoặc cùng lẻ

Mà từ cách xây dựng tập $A$ nên ta suy ra trong tập $A$ có ít nhất một số là số lẻ

Suy ra $b_{1},b_{2},...,b_{n}$ cùng lẻ

Nếu $n$ là số chẵn thì khi bỏ một phần tử ra khỏi tập $A$ thì tập còn lại sẽ có một số lẻ số phần tử, từ đó khi phân hoạch thành hai tập thì tổng các phần tử của mỗi tập sẽ có tính chẵn lẻ khác nhau nên không thể bằng nhau.

Suy ra $n\neq 4,n\neq 6$

Xét $n=5$, khi đó tập $A=\left \{ b_{1},b_{2},b_{3},b_{4},b_{5} \right \}$

Không mất tính tổng quát, giả sử $b_{1}<b_{2}<b_{3}<b_{4}<b_{5}$

Khi bỏ $b_{5}$ khỏi tập $A$ thì $\begin{bmatrix} b_{2}+b_{3}=b_{1}+b_{4}& \\ b_{1}+b_{2}+b_{3}=b_{4}& \end{bmatrix} \Leftrightarrow \begin{bmatrix} b_{2}+b_{3}=b_{1}+b_{4}& \\ b_{2}+b_{3}=b_{4}-b_{1}& \end{bmatrix}$

Khi bỏ $b_{4}$ ra khỏi tập $A$ thì $\begin{bmatrix} b_{2}+b_{3}=b_{1}+b_{5}& \\ b_{1}+b_{2}+b_{3}=b_{5}& \end{bmatrix} \Leftrightarrow \begin{bmatrix} b_{2}+b_{3}=b_{1}+b_{5}& \\ b_{2}+b_{3}=b_{5}-b_{1}& \end{bmatrix}$

Ta thấy ngay vô lí

Suy ra $n\neq 5$

Suy ra $n\geq 7$

Ta xét tập $B=\left \{ 1;3;5;7;9;11;13 \right \}$

Khi bỏ $1$: $3+5+7+9=11+13$

Khi bỏ $3$: $5+7+11=1+9+13$

Khi bỏ $5$: $1+3+7+11=9+13$

Khi bỏ $7$: $3+5+13=1+9+11$

Khi bỏ $9$: $1+3+5+11=7+13$

Khi bỏ $11$: $1+5+13=3+7+9$

Khi bỏ $13$: $2+3+5+9=11+7+1$

Từ đó, ta thấy tập $B$ thõa mãn yêu cầu đề bài

Vậy giá trị nhỏ nhất của $n$ là $7$




#529290 Chọn đội tuyển QG tỉnh Gia Lai 2014-2015

Đã gửi bởi shinichigl on 17-10-2014 - 22:41 trong Thi HSG cấp Tỉnh, Thành phố. Olympic 30-4. Đề thi và kiểm tra đội tuyển các cấp.

đề này là đề 2013-2014 bạn nhé, chứ tỉnh mình chưa thi quốc gia




#526731 Chứng minh AK vuông góc BC

Đã gửi bởi shinichigl on 30-09-2014 - 21:52 trong Hình học

Cho tam giác ABC có các điểm P,Q thuộc cạnh BC sao cho AP=AQ. Đường tròn ngoại tiếp tam giác APB cắt CA tại E khác A. Đường tròn ngoại tiếp tam giác AQC cắt AB tại F khác A. Lấy các điểm M,N lần lượt thuộc tia đối của các tia PE,QF sao cho PM.QN=PE.QF. R là giao điểm của BN và CM. K là tâm đường tròn ngoại tiếp tam giác RMN. Chứng minh rằng AK vuông góc BC.




#524682 Chọn đội tuyển vòng trường 2014 THPT Chuyên LTV, Đồng Nai

Đã gửi bởi shinichigl on 15-09-2014 - 19:03 trong Thi HSG cấp Tỉnh, Thành phố. Olympic 30-4. Đề thi và kiểm tra đội tuyển các cấp.

(1) $\overset{y=0}{\Rightarrow}f\left(f(x)\right)+f(x)=2x,\ \forall x$ (2)

Nếu $x_1,x_2\in\mathbb{R}$ thoả $f(x_1)=f(x_2)\overset{(2)}{\Rightarrow}x_1=x_2\Rightarrow f$ đơn ánh. (3)

 

(1) $\overset{y=\frac{f(x)-x}{2}}{\Rightarrow}f\left(\frac{f(x)+x}{2}\right)=x,\ \forall x$ (4)

$\Rightarrow \forall x,\ \exists z=\frac{f(x)+x}{2}\ :\ f(z)=x$   $\Rightarrow f$ toàn ánh. (5)

 

(3)(5) $\Rightarrow f$ song ánh. (6)

 

(1)(4) $\Rightarrow f(f(x)-y)+f(x+y)=2.f\left(\frac{f(x)+x}{2}\right),\ \forall x,y$ (7)

 

$\forall u,v\ :$ chọn $x=f\left(\frac{u+v}{2}\right)\ ;\ y=v-f\left(\frac{u+v}{2}\right)$ thì $f(x)-y=f\left[f\left(\frac{u+v}{2}\right)\right]+f\left(\frac{u+v}{2}\right)-v\overset{(2)}{=}u+v-v=u$  ;  $x+y=v$ (8)

 

(7)(8) $\Rightarrow f(u)+f(v)=2.f\left(\frac{u+v}{2}\right),\ \forall u,v$ (9)

Từ (6)(9) suy ra $f(x)$ có dạng : $f(x)=ax+b,\ \forall x$. Thay vào (1) ta được $\begin{cases}a=1\\b=0\end{cases}$  hoặc  $\begin{cases}a=-2\\b\in\mathbb{R}\end{cases}$

 

Vậy $f(x)=x\ ,\ \forall x$   hoặc   $f(x)=-2x+b\ ,\ \forall x$ ($b$ hằng số bất kì).

 

------------------------------------------------------------------------------------------------------

 

$\boxed{\text{Cách 2}}$

(1) $\overset{y=0}{\Rightarrow}f\left(f(x)\right)+f(x)=2x,\ \forall x$ (2)

Thay $x$ bởi $f_n(x)=\underset{\text{n lần}}{\underbrace{f(f(...f(x)}}$ vào (2) ta có : $f_{n+2}(x)+f_{n+1}(x)=2.f_n(x),\ \forall x\in\mathbb{R},\ \forall n\in\mathbb{N}$

Đặt $x_n=f_n(x)\ (n\in\mathbb{N})$ ta có pt sai phân : $x_{n+2}+x_{n+1}=2.x_n$

Pt đặc trưng : $\lambda^2+\lambda=2\Leftrightarrow \lambda=1\ \vee\ \lambda=-2$

Suy ra : $x_n=C_1+C_2.(-2)^n$ ($C_1,C_2$ hằng số)

Ta có : $\begin{cases}x=x_0=C_1+C_2\\f(x)=x_1=C_1-2C_2\end{cases}\Rightarrow\begin{cases}x-f(x)=3.C_2\\2x+f(x)=2.C_1\end{cases}$

Suy ra $f(x)=x-3.C_2$   hoặc  $f(x)=-2x+2.C_1$.

Thay lần lượt mỗi hàm số trên vào (1), ta có : $C_2=0$  ;   $C_1$ tùy ý.

Vậy $f(x)=x,\ \forall x$   hoặc  $f(x)=-2x+C$ (với $C$ hằng số bất kì).

Tại sao lại có cái này hả bạn




#524681 Chọn đội tuyển vòng trường 2014 THPT Chuyên LTV, Đồng Nai

Đã gửi bởi shinichigl on 15-09-2014 - 19:00 trong Thi HSG cấp Tỉnh, Thành phố. Olympic 30-4. Đề thi và kiểm tra đội tuyển các cấp.

 

(1) $\overset{y=0}{\Rightarrow}f\left(f(x)\right)+f(x)=2x,\ \forall x$ (2)

Nếu $x_1,x_2\in\mathbb{R}$ thoả $f(x_1)=f(x_2)\overset{(2)}{\Rightarrow}x_1=x_2\Rightarrow f$ đơn ánh. (3)

 

(1) $\overset{y=\frac{f(x)-x}{2}}{\Rightarrow}f\left(\frac{f(x)+x}{2}\right)=x,\ \forall x$ (4)

$\Rightarrow \forall x,\ \exists z=\frac{f(x)+x}{2}\ :\ f(z)=x$   $\Rightarrow f$ toàn ánh. (5)

 

(3)(5) $\Rightarrow f$ song ánh. (6)

 

(1)(4) $\Rightarrow f(f(x)-y)+f(x+y)=2.f\left(\frac{f(x)+x}{2}\right),\ \forall x,y$ (7)

 

$\forall u,v\ :$ chọn $x=f\left(\frac{u+v}{2}\right)\ ;\ y=v-f\left(\frac{u+v}{2}\right)$ thì $f(x)-y=f\left[f\left(\frac{u+v}{2}\right)\right]+f\left(\frac{u+v}{2}\right)-v\overset{(2)}{=}u+v-v=u$  ;  $x+y=v$ (8)

 

(7)(8) $\Rightarrow f(u)+f(v)=2.f\left(\frac{u+v}{2}\right),\ \forall u,v$ (9)

Từ (6)(9) suy ra $f(x)$ có dạng : $f(x)=ax+b,\ \forall x$. Thay vào (1) ta được $\begin{cases}a=1\\b=0\end{cases}$  hoặc  $\begin{cases}a=-2\\b\in\mathbb{R}\end{cases}$

 

Vậy $f(x)=x\ ,\ \forall x$   hoặc   $f(x)=-2x+b\ ,\ \forall x$ ($b$ hằng số bất kì).

 

------------------------------------------------------------------------------------------------------

 

$\boxed{\text{Cách 2}}$

(1) $\overset{y=0}{\Rightarrow}f\left(f(x)\right)+f(x)=2x,\ \forall x$ (2)

Thay $x$ bởi $f_n(x)=\underset{\text{n lần}}{\underbrace{f(f(...f(x)}}$ vào (2) ta có : $f_{n+2}(x)+f_{n+1}(x)=2.f_n(x),\ \forall x\in\mathbb{R},\ \forall n\in\mathbb{N}$

Đặt $x_n=f_n(x)\ (n\in\mathbb{N})$ ta có pt sai phân : $x_{n+2}+x_{n+1}=2.x_n$

Pt đặc trưng : $\lambda^2+\lambda=2\Leftrightarrow \lambda=1\ \vee\ \lambda=-2$

Suy ra : $x_n=C_1+C_2.(-2)^n$ ($C_1,C_2$ hằng số)

Ta có : $\begin{cases}x=x_0=C_1+C_2\\f(x)=x_1=C_1-2C_2\end{cases}\Rightarrow\begin{cases}x-f(x)=3.C_2\\2x+f(x)=2.C_1\end{cases}$

Suy ra $f(x)=x-3.C_2$   hoặc  $f(x)=-2x+2.C_1$.

Thay lần lượt mỗi hàm số trên vào (1), ta có : $C_2=0$  ;   $C_1$ tùy ý.

Vậy $f(x)=x,\ \forall x$   hoặc  $f(x)=-2x+C$ (với $C$ hằng số bất kì).

 

 

bạn giải thích chỗ này giúp mình với




#523941 $$f(x+yf(x))=f(x)+xf(y); \forall x,y \in \mathbb...

Đã gửi bởi shinichigl on 11-09-2014 - 19:47 trong Phương trình hàm

Mình có thắc mắc này : Tại sao phải xét $f(1)=1$ và $f(1)$ khác $1$ mà tại sao không xét giá trị khác ?

Bạn có thể giải thích ở dòng bôi đỏ về   tập hợp của các biến được không ? Và tại sao  $ \frac{y}{x}=f\left ( \frac{y}{x} \right ),\forall y\in \mathbb{Z},x\in \mathbb{N}^{*}$

Mình trả lời theo thứ tự các ý bạn hỏi

- do đề bài yêu cầu liên tục nên mình nghĩ là sẽ quy nạp từ số nguyên đến số hữu tỉ rồi đến số thực, từ đó mình chứng minh f(n)=n trên tập số nguyên, mà chứng minh cái này thì cần tính f(1), từ đó mình tìm cách tính f(1) thì nó phân ra hai trường hợp như trên.

- để quy nạp lên tập hữu tỉ thì chỉ cần $y\in \mathbb{Z},x\in \mathbb{N}^{*}$ thôi

- ta có

$f\left ( x+y \right )=f\left ( x \right )+xf\left ( \frac{y}{f\left ( x \right )} \right ),\forall y\in \mathbb{Z},x\in \mathbb{N}^{*}$

$f\left ( x+y \right )=f\left ( x \right )+y,\forall y\in \mathbb{Z},x\in \mathbb{N}^{*}$

từ hai cái này thì suy ra được $f\left ( \frac{y}{f\left ( x \right )} \right )=\frac{y}{x},\forall y\in \mathbb{Z},x\in \mathbb{N}^{*}$,

do ở trên ta đã chứng minh được $f\left ( n \right )=n,\forall n\in \mathbb{Z}$ nên ta cũng có được

$\Rightarrow f\left ( \frac{y}{f\left ( x \right )} \right )=f\left ( \frac{y}{x} \right )=\frac{y}{x},\forall y\in \mathbb{Z},x\in \mathbb{N}^{*}$

:icon6:




#523911 cho A=$ \begin{Bmatrix} 1,2,3,,....,2014 \end{B...

Đã gửi bởi shinichigl on 11-09-2014 - 15:48 trong Tổ hợp và rời rạc

cho A=$ \begin{Bmatrix} 1,2,3,,....,2014 \end{Bmatrix}$

Có bao nhiêu cặp tập con ($ A_{1}$,$ A_{2}$) sao cho $ A_{1}\bigcup A_{2}=A$

Cặp tập con $\left ( A_{1},A_{2} \right )$ có thứ tự hay không thứ tự hả bạn?

Mình làm trường hợp không có thứ tự

Ta tổng quát bài toán:

Cho A=$ \begin{Bmatrix} 1,2,3,,....,n \end{Bmatrix}$

Có bao nhiêu cặp tập con ($ A_{1}$,$ A_{2}$) sao cho $ A_{1}\bigcup A_{2}=A$

 

Ta sẽ đếm số cách xếp mỗi phần tử $x\in A$ vào hai tập con $B$ và $C$ bất kì của $A$

Rõ ràng có 4 cách để xếp

i) $x$ thuộc $B$ nhưng không thuộc $C$;

ii) $x$ thuộc $C$ nhưng không thuộc $B$;

iii) $x$ thuộc cả $B$ và $C$;

iiii) $x$ không thuộc cả $B$ và $C$,

Xét hai tập con $ A_{1}$, $ A_{2}$ của $A$ thõa mãn bài toán

Theo đề bài thì $ A_{1}\bigcup A_{2}=A$, nghĩa là theo cách xếp như trên thì ta không tính trường hợp iiii), từ đó ta có 3 cách chọn

Mặt khác, do không kể thứ tự cặp ($ A_{1}$,$ A_{2}$) nên có sự đối xứng giữa $ A_{1}$,$ A_{2}$ (tức là cách chọn $x$ thuộc $ A_{1}$ nhưng không thuộc $ A_{2}$ và cách chọn $x$ thuộc $ A_{2}$ nhưng không thuộc $ A_{1}$ là giống nhau ). Chú ý thêm có một trường hợp đặc biệt là khi $A_{1}=A_{2}$ thì phải có $A_{1}=A_{2}=A$ nên chỉ có một cách chọn. Ta bỏ trường hợp này ra, chia đôi các trường hợp còn lại rồi cộng trường hợp này vào thì có số cách sắp xếp là $\frac{3^{n}-1}{2}+1=\frac{3^{n}+1}{2}$




#523886 $$f(x+yf(x))=f(x)+xf(y); \forall x,y \in \mathbb...

Đã gửi bởi shinichigl on 11-09-2014 - 12:23 trong Phương trình hàm

Bài 2:Tìm tất cả hàm số liên tục $\mathbb{R}\rightarrow \mathbb{R}$ thỏa mãn $f(x+yf(x))=f(x)+xf(y); \forall x,y \in \mathbb{R}$ (1)

TH1. $f\left ( 1 \right )\neq 1$

Thay $x=1$, $y=\frac{1}{1-f\left ( 1 \right )}$ vào (1): 

$f\left ( 1+\frac{f\left ( 1 \right )}{1-f\left ( 1 \right )} \right )=f\left ( 1 \right )+f\left ( \frac{1}{1-f\left ( 1 \right )} \right )\Rightarrow f\left ( 1 \right )=0$

Thay $x=1$ vào (1):

$f\left ( 1 \right )=f\left ( 1 \right )+f\left ( y \right ),\forall y\in \mathbb{R}$

$\Rightarrow f\left ( y \right )=0,\forall y\in \mathbb{R}$ (thõa mãn $f$ liên tục)

TH2. $f\left ( 1 \right )=1$

Thay $y=0$ vào (1):

$f\left ( x \right )=f\left ( x \right )+xf\left ( 0 \right ),\forall x\in \mathbb{R}$

$\Rightarrow f\left ( 0 \right )=0$

Thay $x=1$ vào (1):

$f\left ( y+1 \right )=f\left ( y \right )+1,\forall y\in \mathbb{R}$

Bằng quy nạp, ta dễ dàng chứng minh được: $f\left ( y+n \right )=f\left ( y \right )+n,\forall y\in \mathbb{R},n\in \mathbb{Z}$ (2)

Thay $y=0$ vào (2): $f\left ( n \right )=n,\forall n\in \mathbb{Z}$

Thay y bởi $\frac{y}{f\left ( x \right )}$ vào (1): $f\left ( x+y \right )=f\left ( x \right )+xf\left ( \frac{y}{f\left ( x \right )} \right ),\forall y\in \mathbb{Z},x\in \mathbb{N}^{*}$

Từ (2) $\Rightarrow f\left ( x+y \right )=f\left ( x \right )+y,\forall y\in \mathbb{Z},x\in \mathbb{N}^{*}$

$\Rightarrow f\left ( \frac{y}{f\left ( x \right )} \right )=\frac{y}{x}=f\left ( \frac{y}{x} \right ),\forall y\in \mathbb{Z},x\in \mathbb{N}^{*}$

$\Rightarrow f\left ( r \right )=r,\forall r\in \mathbb{Q}$

Với mỗi $x\in \mathbb{R}$ tồn tại dãy số hữu tỉ $\left \{ r_{n} \right \}_{n=1}^{+\infty }$ sao cho $\lim_{n\rightarrow +\infty }r_{n}=x$.

Vì $f$ liên tục nên

$f\left ( x \right )=f\left ( \lim_{n\rightarrow +\infty }r_{n} \right )=\lim_{n\rightarrow +\infty }f\left ( r_{n} \right )=\lim_{n\rightarrow +\infty }r_{n}=x$

$\Rightarrow f\left ( x \right )=x,\forall x\in \mathbb{R}$

Vậy có hai hàm thõa mãn yêu cầu đề bài là

$f\left ( x \right )=0,\forall x\in \mathbb{R}$

$f\left ( x \right )=x,\forall x\in \mathbb{R}$